Pre-Exam 2022: our answers to the claims analysis questions

The first claims part was about yoga mats and textile bags for completely covering and for transporting yoga mats. The second claims part was about sethod for preserving wine, or -more general- alcoholic beverages, in a bottle.

As in the legal part, when you check your answers with ours, note that the order of the statements within a question may be different!

Please feel invited to comment!

During the exam, candidates had access to the EPO Legal Text pages, so including the full EPC Articles and Rules, Guidelines, GL/PCT-EPO, National Law Tables, Case Law, and the Euro-PCT Guide (HTML versions), but not to the PCT Legal Texts on www.wipo.int/pct/en. Access was to the live versions, so to the versions in force on 18 March 2022 (so not the version of 31.10.2021 acc REE/IPEE).

Candidates could access the exam in all languages, English, French and German.

The exam had a mix of common and less-common claims analysis topics.

Please do not post your comments anonymously - it is allowed, but it makes responding more difficult and rather clumsy ("Dear Mr/Mrs/Ms Anonymous of 18-03-2022 22:23"), whereas using your real name or a nickname is more personal, more interesting and makes a more attractive conversation. You do not need to log in or make an account - it is OK to just put your (nick) name at the end of your post.

Note: the legal part is discussed in our other blog post: here

First impressions and general comments can be posted here.

Part 3

Question 11

  1. The features of claim 11 imply a limitation of the dimensions of the textile bag.
  2. A yoga mat consisting of poly-X is covered by the scope of claim 7.
  3. The material of claim 5 is defined by means of a functional feature.
  4. The subject-matter of claim 13 lacks clarity.

Our answers:

  1. T – [001]: yoga mats have a standardized size – so the bag shall not be too small to cover such mat – GL F-IV, 4.14.2 covering sheet for …
  2. F – in view of “consist”
  3. T – “providing …”
  4. T – the organic material is growing in an active way (even after a bad has been made from it?) & relatively slow growth has no clearly defined meaning

Question 12

  1. The subject-matter of claim 2 is novel over D3.
  2. A valid argument that the subject-matter of claim 4 is novel over D3 is that in D3 the first face is the face that is in contact with the athlete.
  3. A valid argument that the subject-matter of claim 6 is novel over D2 is that in D2 the yoga mat is only partially coated with a layer of poly-X.
  4. The subject-matter of claim 8 is novel over each of D1, D2, D3 and D4.

Our answers:

  1. F – “has a” does not imply fully coated
  2. T

Question 13

  1. The subject-matter of claim 10 is novel over D1.
  2. The subject-matter of claim 13 is novel over D2.
  3. The subject-matter of claim 10 is novel over D2.
  4. The present application indicates that a yoga mat made of poly-Y is more effective than a yoga mat made of poly-X or poly-Z in reducing sweat generation on the body of the athlete while the athlete is in contact with the yoga mat.

Our answers:

  1. F – poly-Y [005]
  2. T – fully organic
  3. F – poly-Y in claim only preferred; poly-Z satisfied material features
  4. T – [005], SRV of Poly-Y is 100, whereas it is 84 and 93 for Poly-X resp poly-Z

Question 14

  1. A modified single claim 1 which reads “Yoga mat having a first face and a second face, wherein the first face has a coating of poly-X, the poly-X covering 100% of the first face.” would be allowable under Article 123(2) EPC.
  2. A modified single claim 1 which reads “Yoga mat having a first face and a second face, wherein the first face and the second face contain silver ions.” would be allowable under Article 123(2) EPC.
  3. A modified single claim 1 which reads “Yoga mat having a first face and a second face, wherein the yoga mat has an SRV of 90 or more.” would be clear under Article 84 EPC.
  4. A modified single claim 1 which reads “Yoga mat having a first face and a second face, wherein the yoga mat has an SRV of 90 or more.” would be allowable under Article 123(2) EPC.

Our answers:

  1. T – would be 1+5+6+9(max) w/o 5, but that is OK as poly-X (claim 6) is a "material providing secure but reversible adhesion of the mat onto the floor" (claim 5). I.e., the functional feature is limited to the specific poly-X
  2. F – silver only described on second surface, not on both
  3. F – secret protocol – [005]
  4. T - [005], last sentence. But it would not be clear.

Question 15

  1. In D2 and in the present application the material poly-X has essentially the same purpose.
  2. The additional features defined in claim 8, whereby the second face of the yoga mat contains silver ions, has the technical effect of reducing bacteria growth.
  3. The subject-matter of claim 13 solves the objective technical problem of providing an organic, sustainable yoga mat with respect to D1 as closest prior art.
  4. The subject-matter of claim 14 is novel over D1.
Our answers:

  1. T - provides secure but reversible adhesion of the yoga mat onto the floor
  2. T – [004]
  3. F – [006] very fashionable
  4. F – D1 “because it is made entirely of poly-Y” implicitly discloses ‘Use of poly-Y in the production of yoga mats’


Part 4

Question 16
  1. D12 destroys the novelty of the subject-matter of claim II.2.
  2. D12 destroys the novelty of the subject-matter of claim II.1.
  3. D11 destroys the novelty of the subject-matter of claim II.2.
  4. D13 destroys the novelty of the subject-matter of claim II.1.
Our answers:
  1. T – [002] “We have additionally noted that a part of the injected carbon dioxide reaches the headspace of the bottle and pushes the air (and therefore also oxygen) contained therein out of the headspace of the bottle. Moreover, we found that wine can be better preserved when carbon dioxide is injected therein”
  2. T – idem
  3. T – [ 002] “The stopper has a one-way valve through which air and oxygen from within the bottle can be extracted using a vacuum pump”
  4. F – no wine in D13

Question 17
  1. D12 describes that wine can have a chemical reaction with a gas.
  2. D11 destroys the novelty of the subject-matter of claim II.3.
  3. D13 destroys the novelty of the subject-matter of claim II.5.
  4. D12 destroys the novelty of the subject-matter of claim II.4.
Our answers:
  1. T – “When carbon dioxide is injected into wine, a similar chemical reaction occurs as when carbon dioxide is added to water”
  2. F – no direct injection
  3. T – 5(3)(2) – although D13 [002] does not explicitly say “exclusively” nitrogen, there is no indication that it would be a mixture; “some oxygen is removed as well”
  4. F – feature of II.3 not in [002]
Question 18
  1. The subject-matter of claim II.6 can be introduced into the description without contravening Article 123(2) EPC.
  2. The EPO may issue a communication under Rule 62a EPC in the search phase for the present set of claims.
  3. The presence of an inert gas is described in the application documents as an essential feature.
  4. The subject-matter of claim II.5 is clear.
Our answers:
  1. T – claims are part of appl as filed
  2. T – claims 1&2
  3. F – [003] “preferred”
  4. F – due to dependence on II.4
Question 19
  1. Although argon is described as the most preferred inert gas, deleting all references to argon from the claims would be allowable under Article 123(2) EPC.
  2. Amending claim II.2 to recite “Method for preserving an alcoholic beverage having 10 vol.% alcohol or more, wherein the amount of oxygen in the headspace of the bottle is reduced” would be allowable under Article 123 (2) EPC.
  3. Amending claim II.7 to recite “Method for preserving an alcoholic beverage according to claim II.3, wherein the inert gas is a noble gas” would not be allowable under Article 123(2) EPC.
  4. Amending claim II.3 to recite “Method for preserving an alcoholic beverage according to claim II.2, wherein an inert gas is directly injected into the headspace of the bottle to create a gas cap on top of the wine level” would be allowable under Article 123(2) EPC.
Our answers:
  1. T – [003] on inert gasses in general & [004] giving specific examples of inert gasses. Deleting the preferred embodiment is allowed, as still subject-matter remaining (similar to G 2/10 when it would have been written as an amendment to "insert gas but not argon").
  2. T – [001] “or other alcoholic beverages like beer or other beverages having 10 vol.% alcohol or more”. Comment [added 21/3/22]: We consider the (intentional or accidental?) deletion of "in a bottle" in the title of the claim not to violate 123(2) as one could consider that implicit from the "headspace of the bottle" - the amendment does however introduce a clarity problem as "the bottle" has no antecedent.
    Update 15 April 2022: The Examiners' Report confirms that the deletion of "in a bottle" does not violate Art.123(2): " Although the amended claim II.2 omits the expression “in the bottle”, this does not violate the requirements of Article 123(2) EPC. Amended claim II.2 still references a bottle in the second part of this claim with reference to its headspace."
  3. T – noble gas not in application; note that the statement says “not allowable”
  4. F – only with argon [004]
Question 20

Comment: In the copy we had, Q.20 and its the statements did not refer to a specific claim, while there are 9 different claims... and while GL G-VII, 5.1 (esp. first paragraph) defines the CPA relative to the "claimed invention" and not to, e.g. the invention as specified in quite general terms in the opening paragraph [001] of the description of the application.

Update 15 April 2022: The Examiners' Report indicated:
"Since the following sentence was missing on WiseFlow:
“Assume in the following that inventive step has to be assessed in respect of claim II.9.”
the Examination Board decided to award full marks to all candidates for question 20."

Note that claim II.9, when written out fully, reads:
II.9.    Method for preserving an alcoholic beverage [=wine] in a bottle [from claim II.2], wherein
- the amount of oxygen in the headspace of the bottle is reduced [from claim II.2],
- an inert gas is injected directly into the headspace of the bottle [from claim II.3],
- the inert gas is argon [from claim II.7], and
- the alcoholic beverage is wine and the wine is contained in an opened bottle [from claim II.9]
  1. If D12 is selected as the closest prior art, a possible objective technical problem can be formulated as how to preserve an alcoholic beverage by injecting an inert gas directly into the headspace of the bottle.
  2. Assuming that D13 is considered to be the closest prior art, an objective technical problem to be solved may be regarded as how to reduce the amount of gas required to remove air from above the wine level.
  3. One valid argument as to why D11 is not the closest prior art is that D11 does not require the addition of gas.
  4. One valid argument as to why D13 is not the closest prior art is that D13 deals only with the preservation of fish oil.
Our answers:
  1. F – this is not the problem, but rather the solution; Also note that the statement does not refer to a specific claims, while there are 9 different claims… And claim 1 is “Method for preserving wine in a bottle”, so that this OTP would not match claim 1.
  2. F – there is no wine in D13, so also no wine level
    Update 15 April 2022: w.r.t claim II.9, the statement may be argued to be T (as in the Examiners' Report), even though it feels uncomfortable or even incorrect to consider D13 the CPA (it does not relate to wine, and one has to generalize "nitrogen" in D13 to "gas" in the formulation of the CPA). Note that we do not know whether the statement would possibly have be neutralized  individually (if Q.20 would not have been neutralized in full).
  3. F – it is same field and same purpose, and does not add fuzz to the wine, so keep the wine as it is
    Update 15 April 2022: F may be argued for claim II.1 and II.2; but w.r.t claim II.9 which specifies addition of gas (more specifically, inert gas = argon), T may be argued as is dione in the Examiners' Report.
  4. – “[003] Clearly, this method can be applied to many types of liquid, such as perfumes and beer”
We are looking forward to your comments!

Diane, Roel

(c) DeltaPatents


Annex: claim sets and pre-printable material

 

Part 3 Claims Analysis

Claims filed together with the European patent application:

1.    Yoga mat having a first face and a second face.

2.    Yoga mat according to claim 1, wherein the first face and the second face are distinguishable.

3.    Yoga mat according to claim 2, wherein the first face has a first colour and the second face has a second colour different from the first colour.

4.    Yoga mat according to any of claims 1 to 3, wherein, in use, the first face is in contact with the floor and the second face is in contact with the athlete.

5.    Yoga mat according to any of claims 1 to 4, wherein the first face has a coating of material, said material providing secure but reversible adhesion of the mat onto the floor.

6.    Yoga mat according to claim 5, wherein the material is poly-X.

7.    Yoga mat according to any of claims 1 to 6, wherein the second face contains an antibacterial material.

8.    Yoga mat according to any of claims 1 to 6, wherein the second face of the yoga mat contains silver ions.

9.    Yoga mat according to claim 6, wherein the material covers 10 to 100% of the first face.

10.    Yoga mat according to any of claims 1 to 9, wherein the second face contains a material, preferably poly-Y, wherein the material is such that upon contact with the athlete’s body it reduces the generation of sweat on the athlete’s body.

11.    Textile bag for completely covering and for transporting yoga mats.

12.    The textile bag of claim 11, containing the yoga mat according to any of claims 1 to 10.

13.    The textile bag of claim 12, wherein the bag is made from fully organic material which grows relatively slowly.

14.    Use of poly-Y in the production of yoga mats.

 

Documents for part 3

* Description of a European patent application
* Documents D1 to D4

 

Description of a European patent application

[001] It is generally known that yoga mats are used by athletes to perform yoga exercises.
Yoga mats are thin, soft mats which are easily rolled and transported to allow the athlete to
use them in various locations. Once rolled out on the floor, a yoga mat has a length of 1.5
to 2.2 metres and a width of 0.6 to 1.0 metres. Described herein is an improved yoga mat.

[002] In an embodiment, we provide a yoga mat with a first colour on a first face and a
different second colour on the second face. When the yoga mat is rolled out and in use,
the first face is in contact with the floor and the second face is in contact with the athlete.
This allows the athlete to always use the yoga mat such that the same face of the yoga
mat is in contact with the floor.

[003] In an embodiment, the first face of the yoga mat has a coating of material which
provides secure but reversible adhesion of the yoga mat onto the floor. Preferably, the
material is poly-X. The material covers 10 to 100% of the first face.

[004] In an embodiment, the yoga mat contains antibacterial material, such as e.g. silver
ions on the second face, so as to reduce the growth of bacteria.

[005] In an embodiment, the second face contains a material which reduces the
generation of sweat on the athlete’s body. We have found that poly-Y [note: here, 
the German version said poly-; the French version also said poly-Y] is a highly suitable
material, since it provides improved reduction of sweat generation on the body of the
athlete while the athlete is in contact with the mat. Using our new special secret test
protocol we established a sweat reduction value (SRV) of 100 for yoga mats made of
poly-Y. Higher SRVs correspond to a higher level of sweat reduction. For yoga mats made
of poly-X the SRV is 84, and for yoga mats made of poly-Z the SRV is 93. In an
embodiment of the invention the yoga mat has an SRV of 90 or more.

[006] In an embodiment, we provide the yoga mat with a very fashionable, fully organic
textile container made from slow-growing cotton.
 
Update 15 April 2022: 
The Examiners' Report provides the answers to all statements of Part 3, but it also indicates that complete Part 3 was neutralized in view of the Poly-Y/Poly-X error of translation/transcription:
"There is a translation error in the German version [005] of the description, second line. It should read Poly-Y and not Poly-X.
Due to this translation error the Examination Board decided to award full marks to all candidates for questions 11 to 15." 
 

--

Assume in the following that documents D1, D2, D3, and D4 are prior art in accordance
with Art. 54(1) and (2) EPC:
 

Document D1
This document discloses a yoga mat. Before use, the athlete rolls out the yoga mat onto
the floor. Then, the athlete steps on it to perform exercises. After use, the athlete rolls up
the yoga mat and places it in a bag made from plastic or cotton, in order to transport it
from one location to another. The yoga mat is very easy to transport, because it is made
entirely of poly-Y, a very light, synthetic material.

Document D2
This document discloses a yoga mat containing the material poly-Z on at least one face
of the mat. Poly-Z has the following advantage: upon contact with the athlete’s body it
reduces the generation of sweat on the athlete’s body. Advantageously, in a preferred
embodiment, one face of the yoga mat is partially coated with a layer of poly-X. This
material provides a strong adhesion of the yoga mat onto the floor while the athlete
performs exercises. After use, the mat can be easily removed from the floor.

Document D3
Many athletes mark their yoga mats (e.g. by writing their names on one of the faces) to
clearly distinguish the first face of the mat, which is in contact with the athlete, and the
second face, which remains in contact with the floor.
 
Document D4
Silver ions are used as antibacterial materials in many textiles (such as clothes, carpets)
and on polymer material used for covering the floor of gymnastic halls. Antibacterial
materials have the effect that they prevent or at least reduce the growth of bacteria.
Such an effect is only to be found in very special metal materials such as silver ions or
copper ions.

No figures in part 3 (not in application, not in prior art)

 

Part 4 Claims Analysis

Claims filed together with the second European patent application:

II.1.    Method for preserving wine in a bottle, wherein the composition of a gas in the headspace of the bottle is modified.

II.2.    Method for preserving an alcoholic beverage in a bottle, wherein the amount of oxygen in the headspace of the bottle is reduced.

II.3.    Method for preserving an alcoholic beverage according to claim II.2, wherein an inert gas is injected directly into the headspace of the bottle.

II.4.    Method for preserving an alcoholic beverage according to claim II.3, wherein the inert gas is carbon dioxide.

II.5.    Method for preserving an alcoholic beverage according to claim II.3 or II.4, wherein the inert gas is exclusively nitrogen.

II.6.    Method for preserving an alcoholic beverage according to claim II.3, wherein the inert gas is a mixture of carbon dioxide with nitrogen or with argon.

II.7.    Method for preserving an alcoholic beverage according to claim II.3, wherein the inert gas is argon.

II.8.    Method for preserving an alcoholic beverage according to claim II.5, wherein the alcoholic beverage is wine and the wine is contained in an opened bottle.

II.9.    Method for preserving an alcoholic beverage according to claim II.7, wherein the alcoholic beverage is wine and the wine is contained in an opened bottle.

 

Documents for part 4

* Description of a second European patent application
* Document D11
* Document D12
* Document D13

Description of a second European patent application

[001] The present invention aims at preserving wine (or other alcoholic beverages like
beer or other beverages having 10 vol.% alcohol or more) that remains in a bottle once the
bottle has been opened but not all the wine in the bottle has been consumed. Oxygen from
the air that remains in the bottle, in the space between the wine level and the opening of
the bottle (the so-called “headspace”), causes oxidation of the wine. The oxidation of the
wine renders the wine unpleasant in taste.

[002] A device for preserving wine is known from D11. This device consists of a hand
actuated vacuum pump that is used to remove air (and therefore also oxygen) from the
headspace of the opened bottle. A disadvantage of this device is that the volatile aroma
substances included in the wine are also removed by the pumping process. Another
disadvantage is that many hand actuations are necessary to remove sufficient air from the
headspace.
 
[003] The present invention provides a device that instead of creating a vacuum simply
replaces air (and therefore also oxygen) in the headspace with another gas comprising
less oxygen than air. A preferred other gas is an inert gas. This is achieved by injecting the
other gas (e.g. the inert gas) into the headspace. The injected gas pushes the air out of
the headspace.
 
[004] By inert gas we mean a gas that does not undergo any chemical reaction with the
volatile aroma substances included in the wine. Examples of inert gases are carbon
dioxide, nitrogen and argon. Argon is the most preferred inert gas, because it is heavier
than oxygen and heavier than air, so that a layer of argon creates a “gas cap” on top of the
wine level. Even a very small volume of argon is sufficient to create the gas cap and
prevent the oxidation processes.

---

Assume in the following that documents D11, D12, and D13 are prior art in accordance
with Art. 54(1) and (2) EPC:
[Comment: in the German version, the above sentence was missing]


Document D11

[001] This document discloses a vacuum pump that is used to extract air or oxygen from
an opened bottle in order to preserve the wine contained in the opened bottle.
 
[002] The principle is very simple. The opened bottle is closed with a water-tight and 
airtight reusable stopper. The stopper is easily applied onto the neck of the bottle to
hermetically seal the opening of the bottle. The stopper can be easily removed again
from the bottle by hand. The stopper has a one-way valve through which air and oxygen
from within the bottle can be extracted using a vacuum pump. In this way the quality of
the wine in the opened bottle can be preserved for many days.

 

Document D12

[001] Devices for household use are known for introducing carbon dioxide into water.
Carbon dioxide is a gas that is contained in a pressurized capsule within the device.
Once carbon dioxide is released into water, the gas and the water undergo a chemical
reaction to provide carbonic acid, which gives water pleasant taste and fizz. With the
help of such devices sparkling water can be prepared inexpensively from tap water.
 
[002] We have now found that the same devices can be used to inject the carbon
dioxide into wine. When carbon dioxide is injected into wine, a similar chemical reaction
occurs as when carbon dioxide is added to water. Thereby a pleasant fizz is added to
the wine. We have additionally noted that a part of the injected carbon dioxide reaches
the headspace of the bottle and pushes the air (and therefore also oxygen) contained
therein out of the headspace of the bottle. Moreover, we found that wine can be better
preserved when carbon dioxide is injected therein.

 

Document D13

[001] This document discloses an industrial method for preserving liquids. It is known
that fish oil is easily spoiled by oxygen upon storage. The method consists of injecting
nitrogen into the headspace of a bottle filled with fish oil before closing the bottle and
transporting such bottles to the various points of sale.
 
[002] It is known by the skilled person that nitrogen is lighter than oxygen. Therefore, the
nitrogen must be injected just above the level of the fish oil within the bottle. Nitrogen
evacuates through the opening of the bottle, but some oxygen is removed as well. After
injecting a volume of nitrogen equal to 20 times of the volume of the headspace into the
headspace of the bottle, virtually no oxygen can be traced in the headspace of the
bottle.

[003] Clearly, this method can be applied to many types of liquid, such as perfumes and
beer.

 

No figures in part 4 (not in application, not in prior art)

Comments

  1. Thanks for the quick answers!
    Question 11
    "The features of claim 11 imply a limitation of the dimensions of the textile bag."

    I believed it was false since "Textile bag for" is suitable for, so it does not have that limitation. Moreover, "(par 01) yoga mats have a standardized size – so the bag shall not be too small to cover such mat" does not clearly imply a limitation on the size due to the words "not too small".

    Does it make sense?

    ReplyDelete
    Replies
    1. GL F-IV, 4.14.2:
      "4.14.2 Dimensions and/or shape defined by reference to another entity

      It may be allowable to define the dimensions and/or shape of a first entity in an independent claim by general reference to the dimensions and/or corresponding shape of a second entity which is not part of the claimed first entity but is related to it through use. This particularly applies where the size of the second entity is in some way standardised (for example, in the case of a mounting bracket for a vehicle number-plate, where the bracket frame and fixing elements are defined in relation to the outer shape of the number-plate).

      Furthermore, references to second entities which cannot be seen as subject to standardisation may also be sufficiently clear in cases where the skilled person would have little difficulty in inferring the resultant restriction of the scope of protection for the first entity (for example, in the case of a covering sheet for an agricultural round bale, where the length and breadth of the covering sheet and how it is folded are defined by reference to the bale's circumference, width and diameter, see T 455/92). It is neither necessary for such claims to contain the exact dimensions of the second entity, nor do they have to refer to a combination of the first and second entities. Specifying the length, width and/or height of the first entity without reference to the second would lead to an unwarranted restriction of the scope of protection."

      Delete
    2. Any chance that this statement would be neutralized by the exam board?

      Delete
    3. The dimensions of the mat provided in paragraph [001] of the description apply when the yoga mat is rolled out on the floor. However, as described in D1, for the purposes of transportation, the mat is rolled up. As far as I can see there is no indication in the prior art or the description regarding the dimensions of a rolled-up yoga mat. This would depend on how tightly the mat is rolled up. Therefore, I do not see how one can derive any reasonable limitations on the size of the textile bag.

      Delete
    4. But wouldn't the fact that a bag the size of a matchbox would clearly not be suitable for transporting one or more yoga mats be already indicative of a limitation? IMHO limitation also applies with regard to lower limits, i.e. how small such a bag for transporting yoga mats be.

      The description does not need to give any dimensions if the skilled person would have little difficulty inferring the size of a yoga mat or yoga mats to derive at least a lower size limit of the bag, isnt that the gist of T455/92?

      Delete
    5. The problem with this case is that we are discussing a textile bag, not any container. In essence, a textile bag can be tiny and yet stretch to the dimension of the carried object. I recently worked on an application directed to a stretchy pouch that could expand to 400% of its volume. The portability was listed as an advantage. Hence, I wonder if this applies to textiles.

      I also share the views of the comment above, as I wonder what would be the lower limit implied for a rolled-up mat, especially if it was subsequently folded?

      In the case of T 455/92, the length and breadth of the covering sheet and how it is folded are defined by reference to the bale's circumference, width and diameter; the bale does not undergo further modifications that would alter its dimensions. The bale's dimensions are known and fixed.

      Conversely, the mat's dimensions are provided in the specification for a specific configuration, which is not the same when the mat is transported. For example, the rolled-up or folded mat's size is not standardised unless transported in its extended form. And even then, an elastic textile would not be defined by these dimensions.

      These rules certainly do not apply to any container and circumstances. For example, I assume that containers carrying biological samples are surely not included in these rules.

      Delete
    6. The statement said "A limitation", not that the size of the bag is specifically specified. It excludes the size of, e.g., an A5 envelope. That was at least our interpretation of the statement.

      Delete
    7. @ Roel van Woudenberg 21 March 2022 at 09:27: I am not sure about your last statement.

      An A5 sized elasticised bag can stretch and cover even two yoga mats. They do exist in practice by the way. What would then be the intended limitation?

      Delete
    8. That the material and its elasticity are such that it can be stretched to cover it/them.

      Delete
    9. @Roel van Woudenberg 21 March 2022 at 11:43: the question reads that:
      "the features of claim 11 imply a limitation of the dimensions of the textile bag".

      In the case you mentioned above, the material and its elasticity are not limitation of the textile bag's DIMENSION, but different physical attributes.

      The claim is directed to a TEXTILE bag, not a cardboard, metal bag or plastic bag. Why specify textile if it is not to be considered in its broadest sense? The claim could have been directed to cotton specifically, but was purposely drafted broadly.

      Delete
    10. Please use your name or a nickname. Are you the same anonymous as three comments above?

      The limitation would be that, even for an elasticised textile bag can stretch and cover even two yoga mats, the claim specifies that that bag is a bag for completely covering and for transporting yoga mats. If it would be too small, also if stretched, it would not be under the scope of the claim - so, the claim does imply some degree of limitation of the dimensions of the bag, although not in a very clear way.

      But even if not so clear, it does excluded some textile bags because of their dimension: An A5-sized textile bag with the shape of a normal envelope that is not stretchable is clearly not within the scope of the claim, due to its dimension.

      Delete
    11. Sorry, I am indeed the same anonymous of above. I now have a nickname included.

      Please consider the following:
      Claim 11 recites:
      "Textile bag FOR completely covering and for transporting yoga mats".

      GL f-IV-4.14 states that:
      A claim in respect of a physical entity (product, apparatus) may seek to define the invention by reference to features relating to another entity that is not part of the claimed first entity but that is related to it through use. An example of such a claim is "a cylinder head FOR an engine", where the former is defined by features of its location in the latter.

      Since the first entity (the cylinder head) can often be produced and marketed independently of the other entity (the engine), the applicant is normally entitled to independent protection of the first entity per se. Therefore, in first instance, such a claim is always interpreted as not including the other entity or its features: these limit the subject-matter of the claim only in so far as the first entity's features are suitable to be used with the second entity's features. In the above example, the cylinder head must be suitable to be mounted in the engine described in the claim, but the features of the engine do not limit the subject-matter of the claim per se.

      Only if the claim is directed without any doubt to a combination of the first and second entities, the features of the other entity are limiting for the subject-matter of the claim. In the above example, the claim should be written as an "engine with a cylinder head" or an "engine comprising a cylinder head" for the features of the engine to be considered as limiting the subject-matter of the claim.

      Could you please explain why claim 11 doesn't fall within the first example?

      Thanks

      Delete
    12. And to follow up, please also note that GL F-IV-4.14.2 states:
      It may be allowable to define the dimensions and/or shape of a first entity in an independent claim by general reference to the dimensions and/or corresponding shape of a second entity which is not part of the claimed first entity but is related to it through use.

      Claim 11 does not attempt to define the dimension of the textile bag.

      The claim referred to T455/92 recites (this is a translation):

      "Cover for a pressed product (20) produced in a press chamber (28) of a rectangular or round baler, which is open on at least one side, which is fed to the press chamber (28) and wrapped around the pressed product (20), characterized by

      a) a length that corresponds at least to the circumference of the outer surface of the pressed product (20) to be covered,

      b) an overall width that is sufficient to cover the lateral surface and both end faces of the pressed product (20), and

      c) a fold such that the cover is folded over on both sides essentially parallel to the longitudinal center line, so that the width of the base part (12) of the folded cover corresponds to the width of the lateral surface of the pressed product (20) and the width of the folded over Side parts (14) are sufficient to cover the end faces of the pressed product (20)."

      This claim clearly attempts to define the dimension of the cover sheet. Conversely, claim 11 does not even try to define the dimension of the bag and therefore, GL F-IV-4.14 seems more suitable in this case.

      Delete
  2. Suitable for means that the yoga mats needs to fit inside the bag, therefore there is a limitation of the dimension of the bag

    ReplyDelete
    Replies
    1. Was there a clearly defined thickness of the mat?

      Delete
    2. However, I think it could be stated that the wording of claim 11 is not directed to a bag suitable for a (one) yoga mat but for a plurality of mats. Thus, even if a single yoga mat has a certain size, the bag according to claim 11 does not

      Delete
    3. I had not considered that interpretation. I understood “mats” in that claim to cover that the bad is not designed for one specific individual may, but that it can be used today with a first mat and tomorrow with another mat - and as they have standardized size, that “interchange” is not an issue.

      But your interpretation also seems reasonable.

      The claim seems to be unclear, as is it not clear whether my “interchangeable” (sequentially one at a time) or your “plurality” (at the same time) is meant - and what the committee meant.

      Any other arguments or views?

      Delete
    4. Does GL F-IV, 4.14.2 apply to textile bag which can be elasticised (stretchy yoga mat bags exist by the way), and therefore, may not be limited by the size of the mat(s)?

      Delete
  3. question 19.2 - what about the fact that they removed "in a bottle"? Karine

    ReplyDelete
    Replies
    1. I thought that “in a bottle” is included due to the dependency on claim II.2,

      Delete
    2. Claim II.w was filed as an independent claim reading:

      II.2. Method for preserving an alcoholic beverage in a bottle, wherein the amount of oxygen in the headspace of the bottle is reduced.

      and in 19.2, it is amended to:

      Method for preserving an alcoholic beverage having 10 vol.% alcohol or more, wherein the amount of oxygen in the headspace of the bottle is reduced

      i.e., "having 10 vol.% alcohol or more" is added.
      That additional is allowed based on [001]:

      The present invention aims at preserving wine (or other alcoholic beverages like beer or other beverages having 10 vol.% alcohol or more) ...

      The deletion of "in a bottle" seems uninterntional, as the amended claim further refers to "in the headspace of the bottle". It may be challenged whether that would be a 123(2) violation - in my view it could rather be considered a Art.84 violation (which can be remedied under R.139).

      Delete
    3. I have put false because the description says "or other beverages having 10 vol.% alcohol or more" and not other 'alcoholic' beverages having 10 vol.% alcohol or more. I did not think it was supported because the percentage related to other beverages not other 'alcoholic beverages'.

      Although I can now see that as it says 10 vol.% alcohol or more that may be enough support for 'alcoholic beverages'.

      Delete
    4. The phrase "or other alcoholic beverages like beer or other beverages having 10 vol.% alcohol or more” is not clear to me.
      It seems to indicate that beer has 10 vol% or more alcohol, which it usually does not have (except from some specials such as quadruples from Belgium). The usual precentage is more like 5-6.

      Delete
  4. Karine - I got 19.2 'wrong' because the bottle had been deleted from "method for". I spent ages wondering if it was deliberate/accidental... Oh well...

    ReplyDelete
    Replies
    1. See above. I think it is accidental. If not, you could argue both ways - but if you argue that 123(2) is violated, you need to also consider the effect of the presence of the "in the headspace of the bottle" - would that not lift the violation?

      Delete
    2. I was very much in doubt on this question. I would argue that in claim II.2., there is a link between the alcoholic beverage and the bottle. In the amended claim, this link is removed, and the claim now covers embodiments in which the beverage is not located in the bottle. There is no basis for such embodiments in the application. Thoughts?

      Best,
      Nils

      Delete
  5. Any possibility of question 20 being neutralized? In 2021, the last question also included CPA-related sub-questions. But, a specific claim was provided at that time and the Examiner's report showed reasoning based on the specific claim as well.

    ReplyDelete
    Replies
    1. Indeed, poor CPA questions, as not indicated which claim each of statements related with. So question ill-defined. Neutralization seems appropriate, isn’t it?

      Delete
    2. It seems that it was not an artefact of the copy that we got, but a true omission in the paper: no reference to a specific claim in the question body nor in the statements, while there are 9 different claims and CPA can (at least in principle) be different for all 9 claims. Further, the two independent claims have a different claimed object (II.1. Method for preserving wine in a bottle vs II.2. Method for preserving an alcoholic beverage in a bottle), so different specific field, so that there is a priori an expectation that CPA may be different (as the CPA for a golf ball and for a sports ball can be different - for the latter, it cam be a rugby ball, which is unlikely for the first)

      Delete
    3. Do you believe neutralization is appropriate? Thanks.

      Delete
    4. I agree that question 20 was incredibly confusing because of the reasons you've mentioned, in fact it is the only question where I'll get 0 points according to the answers posted above. Those CPA statements should have been based on a specific claim.

      Delete
    5. Should this question be neutralized?

      Delete
  6. Did anyone else consider the method for “preserving wine” to not be a limiting feature of the claim? It seems like a mere technical effect that would inevitably result from the method rather than an actual physical step.

    Guidelines F-IV 4.13.3, with respect to a "method claim" the following is stated: "… where the purpose merely states a technical effect which inevitably arises when carrying out the other remaining steps of the claimed method and is thus inherent in those steps, this technical effect has no limiting effect on the subject-matter of the claim".

    ReplyDelete
    Replies
    1. I had the same thoughts!

      Delete
    2. If this is true, it could be discussed whether question 16.4 is acutally T instead of F

      Delete
    3. I have seen it that way too. For the same reason.

      Delete
    4. I also said T for this reason.

      Delete
    5. The same to me. I considered D4 broad for attacking novelty

      Delete
    6. You may have a point IF the second part of the claim also mention wine, but it does not, so that -in our opinion, but please give arguments to the contrary- the claim if of the first type of "methods for" of F-IV, 4.13.3:

      "Where the stated purpose defines the specific application of the method, this purpose requires additional steps which are not implied by or inherent in the other remaining steps defined in the claim, and without which the claimed process would not achieve the stated purpose. Hence a method claim that defines a working method which, for example, commences with such words as "Method for remelting galvanic layers", the part "for remelting ..." is not to be understood as meaning that the process is merely suitable for remelting galvanic layers, but rather as a functional feature concerning the remelting of galvanic layers and, hence, defining one of the method steps of the claimed working method (see T 1931/14 and T 848/93)."

      Delete
    7. Hi Roel, in my opinion “preserving wine” does not imply a physical step distinct from the remaining features of the method claims as was the case in the “remelting of galvanic layers”. Secondly, the preservation is an intended technical effect that results from the method steps. Thus, I am not entirely certain if the first type that you indicated here fits better than the second type.

      But we will see what the EPO reports.

      Delete
    8. Yes, I interpreted "for preserving wine in a bottle" as "suitable for", i.e. not limited to wine. D13's method is suitable for preserving wine because [003] states that the method can be used for many types of liquid.

      Delete
    9. I think that the claim could be interpreted as being limited to "preserving wine in a bottle" or merely suitable for "preserving wine in a bottle" (i.e. T/F).

      The fact that the term "the bottle" is used in the modifying step instead of "a bottle" implies that we are referring to the bottle with wine in it. As such, it would make sense to interpret the wine as being limiting. Additionally, the term "preserving [by modifying gas composition]" could be interpreted as a step of the method.

      Alternatively, the "wine in a bottle" could be interpreted as being non-limiting. Indeed the preserving "step" could also be interpreted as a technical effect given the use of "[suitable] for" in the claim. This interpretation would make sense as none of the steps are dependent on, or specifically tailored towards, wine and the description mentions other alcoholic beverages.

      I answered that D13 would not destroy novelty because of the antecedent basis of "wine in a bottle". However, I think either answer could be validly argued.

      Delete
    10. I agree. The only thing is that “preserving wine” implies a technical effect because the description makes it clear that their method allows the taste of the wine to be preserved.

      If the claim would have recited “method for storing wine in a bottle” then it would be a limiting feature because it’s an application and an actual physical step while preserving wine is an “advantage” that you hope to achieve by the method step.

      But I guess it is possible to argue both ways!

      Delete
    11. Good discussion. ;)

      I consider the “for preserving wine in a bottle” to imply that 1) wine is in the bottle, 2) the wine is or will be in a closed or re-closed bottle. In that interpretation, it is a limitation - the method does not cover a situation wherein the bottle is open.

      Delete
    12. Interesting interpretation.
      So if someone keeps their used wine bottle without a cap in the fridge, this would be excluded from the claim scope according to your interpretation.

      Delete
    13. IMHO in claim 1, "preserving wine" is limiting feature, as "modifying the gas composition in the head space" also comprises ADDING oxygen. Hence, merely modifying the gas composition does not inevitably lead to preservation of wine.

      Regarding claim 2 its iffy IMHO. Reducying oxygen in the headspace is disclosed to be a preservative step, no matter how such reduction is achieved. Would an argument as to why "preserving an alcoholic beverage" is still a limiting features be that the claimed method does not exclude other steps that could potentially be detrimental to preserving the alcoholic beverage? In other words, in order to render "preserving an alcoholic beverage" not a limitation, would it be required to specify another method step such as "maintaining the reduced amount of oxygen in the head space"?

      Delete
    14. @Asim: I do not see how the method if the invention could be used if the bottle is open (except for the "gas cap" version of [004] - the headspace of the bottle would then be extended to the whole interior volume of your fridge! Also see [003]:
      The present invention provides a device that instead of creating a vacuum simply
      replaces air (and therefore also oxygen) in the headspace with another gas comprising
      less oxygen than air. A preferred other gas is an inert gas. This is achieved by injecting the
      other gas (e.g. the inert gas) into the headspace. The injected gas pushes the air out of
      the headspace.

      Delete
    15. Hi Roel, I don’t think the headspace would be extended to the whole fridge volume because paragraph 001 says:

      “in the space between the wine level and the opening of the bottle (the so-called “headspace”)”

      So I would say an open bottle is covered by the claim scope.

      Delete
  7. For 16.4, did the claim not say "method for preserving wine" or similar? In this case, wouldn't the comparable disclosure in relation to e.g. beer of d13 be enough to destroy Novelty? Maybe I'm misremembering though

    ReplyDelete
    Replies
    1. For lack of novelty, comparable (beer vs wine) it not good enough - that would be a matter of inventive step.

      Delete
  8. Did anyone else consider 14.4 to be an intermediate generalisation?

    ReplyDelete
    Replies
    1. Can you give your argumentation? (We could have overlooked something)

      Delete
    2. I initially had T as my answer for the same reason as the Delta answers, however upon reading through again I changed my mind. The feature of "the yoga mat has an SRV of 90 or more" is only disclosed in combination with the features that 'the second face contains a material which reduces the generation of sweat on the athlete’s body' namely only poly-Y and poly-Z. Therefore it seems like generalising to that SRV value and plucking that feature out to delimit the claim would not be allowed? This feature is inextricably linked to the features of the second face having poly-Y and poly-Z, thus the generalisation is not allowable under 123(2) GL H-V 3.2.1? That was my thinking anyway!

      Delete
    3. I thought the same, i.e. isolating the SRV from the material is an unjustified intermediate generalisation (because SRV depends on the material).

      Delete
    4. @PR: as in many pre-exams, the 123(2) statements are -at least in my view- the most difficult statements in the exam. And they often lead to quite some discussion on the blogs (and among us).

      I consider the functional feature of the material covered by “SRV > 90”, so ok

      [005] In an embodiment, the second face contains a material which reduces the
      generation of sweat on the athlete’s body. We have found that poly-Y is a highly suitable
      material, since it provides improved reduction of sweat generation on the body of the
      athlete while the athlete is in contact with the mat. Using our new special secret test
      protocol we established a sweat reduction value (SRV) of 100 for yoga mats made of
      poly-Y. Higher SRVs correspond to a higher level of sweat reduction. For yoga mats made
      of poly-X the SRV is 84, and for yoga mats made of poly-Z the SRV is 93. In an
      embodiment of the invention the yoga mat has an SRV of 90 or more

      Delete
    5. Any further views and arguments are welcome!

      Delete
    6. I think this one is tricky, I also chose F for not allowed, but upon further reading I am not so sure anymore.

      Par. [0005] discloses (i) Poly-Y has an SVR of 100, (ii) Poly-X that is intended for adhesion to the floow has an SVR of 84, and Poly-Z has an SVR of 93. So it appears that any material may have an SVR value. The last sentence is somewhat ambiguous. If I was the applicant, I would simply argue that it says - expressis verbis - that in an embodiment of the invention, the yoga mat has an SRV of 90 or more, without requiring any specific material. In such a case, Art. 123(2) is not violated, but rather Art. 84 since essential feature would then be missing. An an opponent however, an equal argument could be made that the skilled person reads the application to make technical sense of it and would thus diretly and unambiguously understand the last sentence to be a preferred embodiment in relation the the entire paragraph, i.e. that either Poly-Y and Poly-Z should be used. Upon reflection IMHO the applicant point of view appears stronger to me, at least with regard to Art. 123(2).

      Delete
  9. Question 17

    - "D12 describes that wine can have a chemical reaction with a gas."

    I understand that the genus discloses the species. However, the term "gas" is an impossibly broad term that includes every known chemical compound or element that in a gaseous phase. Surely the skilled person cannot learn from D12 that wine can have a chemical reaction with any/all of these compounds or elements? Answering "T" to the question seems to me to be taking the "species/genus" principle to an almost absurd extent.

    Furthermore, I know that we are not allowed to use any specialised knowledge when answering the questions, but I would argue that it is well-known that in order to have a chemical reaction, certain parameters (e.g., temperature) must be used. Therefore, can D12 really be considered as enably disclosed for gases in general in order to obtain a chemical reaction with wine?

    ReplyDelete
    Replies
    1. From the prior art document: "When carbon dioxide is injected into wine, a similar chemical reaction
      occurs as when carbon dioxide is added to water".

      I think it is crystal clear.

      Delete
    2. My comment was rather directed as to whether there are any limits to applying the "species/genus" principle.

      If one where to claim the use of a gas that undergoes a chemical reaction with the wine, this could potentially be viewed as a reach-through claim. So I was asking whether a similar principle can be applied to prior art disclosure.

      Furthermore, the question asked whether a gas "can" have a chemical reaction. The word "can" merely indicates that there is a possibility of a chemical reaction. It does not indicate that there "will" be a chemical reaction. By contrast, the prior art indicates that there "will" be a chemical reaction.

      Delete
  10. In question 14.1, the functional feature "providing secure but reversible adhesion of the mat onto the floor" was removed. This functional feature is related to poly-X. I would argue that the amended claim is no longer limited to embodiments in which poly-X provides secure but reversible adhesion of the mat onto the floor. The amended claim covers embodiments where poly-X is applied without providing this functionality (e.g. thinner/thicker layer, mixed with another component), i.e. such that it does not provide secure but reversible adhesion of the mat onto the floor. There is no basis for such embodiments in the application. The application even provides a hint that poly-X is usable for other purposes, such as sweat reduction (it has a so-called sweat reduction value (SRV) of 84). Accordingly, I think that the modified claim is not allowable under 123(2) EPC. Your thoughts?

    Best,
    Nils

    ReplyDelete
    Replies
    1. I did think about this for a while during the exam, but the description teaches that poly-X provides that functional feature, so it's arguably superfluous to leave in the functional feature if you narrow down to a specific material that achieves the functional feature.

      Delete
    2. @Nils: the amended claim says “ has a coating of poly-X, the poly-X covering 100% of the first face” - we thus followed the same argumentation as LH

      Delete
  11. For 14-4, doesn't the specification and the paragraph [0005] imply that poly-Y is used in the said embodiment? I've marked false for that one, though it is specifically given as an embodiment, thinking that the combination of poly-Y and the feature regarding SRV was implied to be what the embodiment consists of, thus the isolation of the feature regarding the SRV value should be be considered as intermediate generalisation.

    ReplyDelete
    Replies
    1. We had a discussion on this. It depends on how you read "Using our new special secret test protocol we established a sweat reduction value (SRV) of 100 for yoga mats made of poly-Y." in [005]: that COULD be read as that the secres test protocol is established for, and only words with, yoga mats made of poly-Y.
      However, such an interpretation is not consistent with the next-next sentence: "For yoga mats made of poly-X the SRV is 84, and for yoga mats made of poly-Z the SRV is 93." Thus, it is also possible to establish an SRV for poly-X and poly-Z mats. Hence: they have a special secret test protocol to establish a sweat reduction value (SRV) for yoga mats, with higher SRVs correspond to a higher level of sweat reduction, and the different poly-X/Y/Z mats have a different SRV 84/100/93.

      Delete
    2. Hi Roel,

      We have found that there was a discrepancy between the English and the German version of the descriptions. That is, paragraph [005] of the German version states that "Using our new special secret test protocol we established a sweat reduction value (SRV) of 100 for yoga mats made of poly-X." (instead of Poly-Y). Thus, we are now wondering whether the questions which relate to this special feature have to be neutralized (or whatever) as this difference alters how one would read the invention as according to the German text the material Poly-X has two functions: sweat reduction as well as improved adhesion.

      I would be interested in your opinion regarding this matter.

      Delete
    3. If the difference has an effect on some statements (I did not check to which statements its effect extends), it seems reasonable to expect that those will be neutralized: Appeal D1/16 (DE), D2/16 (DE), D4/16 (EN), D13/16 (DE) and D 15/16 (DE) addressed differences between the English and German version of statement 5.4 of Pre-Exam 2015 and were successful. Refer to the full decisions for the situation and the reasons for the decision, e.g.:
      EN: https://www.epo.org/law-practice/case-law-appeals/recent/d160004eu1.html
      DE: https://www.epo.org/law-practice/case-law-appeals/recent/d160002du1.html

      NB: The discrepancy was already noted, it is also indicated in the blog post.

      Delete
  12. Firstly, intermediate generalization is allowed in some cases (see Guidelines, H-V 3.2.1). So the mere fact that it is intermediate generalization does not violate Art. 123(2) EPC, unless a skilled person has a clear reason to believe that the feature omitted was inextricably linked to the remaining part.
    In this regard, a skilled person taking the information from the application as a whole would understand that poly-X would not be used for sweat reduction since the SRV value is 84 and they mention 90 or more is the range that the material should have to provide the sweat reduction function. Thus, as LH mentioned, reciting the function of poly-X in the claim would be redundant.

    ReplyDelete
    Replies
    1. This comment has been removed by the author.

      Delete
    2. Inclusion of poly-X would indeed be redundant as its SRV is below 90, however I think that the specification as a whole implies that poly-Y and poly-Z are inextricably linked to the SRV value, because said value directly depends on the material chosen for the purpose and the disclosure does not justify the generalising isolation of the feature regarding the SRV value. Thus, it should not be possible to isolate the SRV value and amend the claim in such a way without offending 123(2).

      The disclosure states that the inventor has established a sweat reduction value (SRV) of 100 for yoga mats made of poly-Y and a sweat reduction value (SRV) of 93 for yoga mats made of poly-Z, so this implies that these values are only achievable when the yoga mat entirely consists of said materials since it's clearly stated that these values are only possible when the yoga mat is MADE OF one of the said materials. The yoga mat could also contain other materials in combination with poly-Y or poly-Z, or a combination of poly-X and poly-Y, thus reducing the overall SRV value while keeping it above 90, and this embodiment only makes sense in that regard. The claim as amended covers all yoga mats with an SRV value above 90 and these would obviously include yoga mats made of other materials, for example poly-N, and I don't see how this would not be considered as intermediate generalisation. In my opinion the claim as amended would have to read "Yoga mat having a first face and a second face, wherein the yoga mat comprises poly-Y or poly-Z and has an SRV of 90 or more." to not offend 123(2).

      Delete
    3. I would rather understand "In an embodiment of the invention the yoga mat has an SRV of 90 or more" (last sentence of [005]) to be applicable to the type of embodiment described in [005[, of which the opening sentence gives the generic feature that is needed when using an SRV: " In an embodiment, the second face contains a material which reduces the generation of sweat on the athlete’s body." The contains in this sentence includes "(fully) composed of" we well as "having a coating on one of its faces" and other "comprises" embodiments as long as one face has a high-SRV material.

      Delete
    4. That is also an issue, as only one face is supposed to have a material with a value of SRV but this is disregarded in [0005]

      Delete
  13. The question 13.4 is very unclear.

    In the description, it’s indicated that "In an embodiment of the invention the yoga mat has an SRV of 90 or more". There is no indication in the description that 100 is the maximal limit for a SRV.
    So in this embodiment, the SRV can have a SRV of 130, 150 or more. So maybe we can reach a SRV of 150 with again poly-Z with this embodiment. And the last words of the previous sentence is about poly-Z.

    Therefore, we can’t conclude that : "The present application indicates that a yoga mat made of poly-Y is more effective than a yoga mat made of poly-X or poly-Z in reducing sweat generation on the body of the athlete while the athlete is in contact with the yoga mat."

    With this point of view, the answer is False.

    But the question is very unclear and I think this question should be Neutralized.

    ReplyDelete
    Replies
    1. It does not matter if SRV has a maximum value of 100 or not (130,150, or more).The value 100 is always higher than 93, right?

      Then the question and the answer are clear.

      Delete
    2. But the embodiment with a SRV of 90 or more can (maybe) be with poly-Z. So maybe we can reach a SRV of 150 with again poly-Z with this embodiment. And 150>100.

      Therefore, we can’t conclude that: "The present application indicates that a yoga mat made of poly-Y is more effective than a yoga mat made of poly-X or poly-Z in reducing sweat generation on the body of the athlete while the athlete is in contact with the yoga mat."

      Delete
    3. Maybe? Rule 22(3) IPREE requires you to "Candidates shall accept the facts given in the examination paper and limit themselves to those facts. Whether and to what extent those facts are used shall be the responsibility of each candidate. Candidates shall not use any special knowledge they may have of the technical field of the invention."

      As the paper clearly says, in [005]:
      "[...] we established a sweat reduction value (SRV) of 100 for yoga mats made of poly-Y" and
      "For yoga mats made of poly-X the SRV is 84," and
      "for yoga mats made of poly-Z the SRV is 93",
      and 100 > 84,
      and 100 > 93,
      I do not see any room for your speculated SRV of 150.

      Delete
    4. But we have a yoga mat made with either poly-Y, poly-X or poly-Z.
      And it's indicated that in an embodiment, the SRV is 90 or more with no indication if it's with poly-Y, poly-X or poly-Z.

      So we can't make any conclusion to say that poly-Y>poly-Z.

      Delete
    5. It is explicitly stated that a yoga mat made of poly-Y has an SRV of 100, while Z has 93 and X has 84. There is no information regarding the embodiment you are referring to and I think we were not required to take that embodiment into consideration to reach a conclusion for this particular question. A yoga mat, for example, could contain poly-X in combination with other materials and have an SRV value of 95 but the question's wording is exactly the same as the passage given in the specification, so it should be clear that a yoga mat MADE OF poly-Y is more effective than a yoga mat made of poly-X or poly-Z.

      Delete
  14. Question 18.2: Can one argue that a communication under R62a can not be issues because claims 1 and 2 fall within the exception of R43(2)C (c): alternative solutions to a particular problem, where it is inappropriate to cover these alternatives by a single claim?

    ReplyDelete
    Replies
    1. IMHO it would be difficult. Please consider the following claims:

      Claim 1: Preserve alcoholic beverages by altering gas composition in head space.

      Claim 2 according to claim 1, wherein the alcoholic beverage is a wine and altering the gas composition comprises removing oxygen from the headspace.

      It would appear to me that such a claim would not contravene Art. 123(2) and would also be appropriate.

      Delete
    2. IMHO, claims 1 and 2 cover alternatives that cannot be combined in a single claim (in contrast to being made dependent to one another).

      Claim 2 covers methods in which the gas in the head space is completely removed or vacuumed (like in D11), therefore, no gas is left in the head space.

      Claim 1 suggest to alter the gas composition (which includes in which the oxygen is not reduced, but maybe a small layer of argon is added, and without reducing the oxygen, simply creates a gas cap).

      I believe that we should read the question as is there a basis to issue a communication under R62a?

      Delete
    3. AFAIK "alternative" solutions means mutually exclusive solutions, which in the above case, they are not IMHO. The claims can be altered within the requirements of Art 123(2) EPC ,i.e. within what the invention discloses, they do not have to be kept the same no?

      Delete
    4. I would have to disagree with Vulgor, because the given claims would have a different meaning and scope of protection. Claim 1 and 2 state that "Method for preserving wine in a bottle, wherein the composition of a gas in the headspace of the bottle is modified." and "Method for preserving an alcoholic beverage in a bottle, wherein the amount of oxygen in the headspace of the bottle is reduced." respectively. So what Vulgor said would make sense if wine and alcoholic beverage swapped places, because the wine case would simply become a subset and an embodiment of the alcoholic beverage case since wine is an alcoholic beverage and reducing the amount of oxygen in the headspace is merely modifying the composition of the gas therein. Thus you could change claim 2 to make it a dependent claim. In the current case however, they appear to be mutually exclusive.

      Delete
    5. @ Salih

      You do have a point. It was my understanding of the case law that, if possible within the extent of Art. 123(2), the applicant is required to specify a single claim using more generalized language/features, but I may be completely wrong. In this case it does appear to be difficult.

      However, would claim 1 and claim 2 not have to solve the SAME problem? At least that's my understanding of Rule 43(2)(c). IMHO, merely changing the gas composition does not solve the same "particular" problem as reducing the amount of oxygen. Hence the exception wouldn't apply, no?

      Delete
    6. I agree with what you said and it is indeed difficult to draft a claim that would include all features of claim 1 and claim 2 in a generalized manner.

      Looking at F-IV, 3.2 of the guidelines, I think these two could be interpreted as alternative solutions to a particular problem because the composition of the gas in the bottle could be modified without reducing the amount of oxygen therein. In fact, injecting, for example, CO2 into the headspace to replace the air would actually increase the amount of oxygen (it would not be more o2 but still more oxygen atoms would be there) in the headspace of the bottle because air is mostly composed of nitrogen anyway. This obviously contradicts the teaching in the specification ([0003] in particular) and might be an oversight but I'm not sure if we were to take the specification into account for this particular question. The right answer to the question about 62a is probably true but it's very confusing regardless.

      Delete
    7. Rule 62a(1)/43(2), and especially 43(2)(c) is always difficult. We answered T because the claims, in the way they are presented, seem to be independent claims (even though, if one considers the proposed rephrasing by Vulgor 20 March 2022 at 22:15 above, they could be considered dependent) AND because the statement says "MAY" and not "WILL" - the EPO may based on a priori consideration consider them to violdate 43((2) . As indicated in B-VIII, 4.2.2, the applicant can give arguments why he/she considers the claims to satisfy 43(2).
      I am ware that the MAY/WILL point is maybe not so convincing, but Rule 62a uses SHALL - the phrasing of the statement may thus br a hint to "not think to deeply as to whether such objection will really be justified.

      Rule 62a Applications containing a plurality of independent claims
      (1) If the European Patent Office considers that the claims as filed do not comply with Rule 43, paragraph 2, it SHALL invite the applicant to indicate, within a period of two months, the claims complying with Rule 43, paragraph 2, on the basis of which the search is to be carried out. If the applicant fails to provide such an indication in due time, the search shall be carried out on the basis of the first claim in each category.
      (2) The Examining Division shall invite the applicant to restrict the claims to the subject-matter searched UNLESS it finds that the objection under paragraph 1 was NOT JUSTIFIED [e.g., after having considered R.70a response].

      Delete
    8. NB: GL B-VIII, 4.1 uses the term MAY rather than SHALL, in that case (in my understanding) because they can also give the benefit of the doubt to the applicant and decide to search all claims:

      4.1 Invitation to indicate which independent claim to search
      If the European Patent Office considers that the claims as filed do not comply with Rule 43(2) (see F‑IV, 3.2), it MAY invite the applicant to indicate, within a period of two months, claims complying with Rule 43(2) on the basis of which the search is to be carried out. Along the lines of Rule 64, the search division has the DISCRETION EITHER to send this invitation OR to make a complete search for all claims, raising the objection under Rule 43(2) only in the written opinion.

      Delete
    9. @ Roel van Woudenberg, as you pointed out above, the EPO will issue a communication under R 62(a) EPC, only if it considers that the claims as filed do not comply with Rule 43(2).

      Therefore, do you consider that the two independent claims do not comply with Rule 43(2)?

      Delete
    10. Yes, I do:

      II.1. Method for preserving wine in a bottle, wherein the composition of a gas in the headspace of the bottle is modified.

      II.2. Method for preserving an alcoholic beverage in a bottle, wherein the amount of oxygen in the headspace of the bottle is reduced.

      II.1 and II.2 are independent, as you cannot consider II.1 a species of II.2 nor the other way round: the preamble of II.1 is more specific (wine vs alcoholic beverage), whereas the wherein part of II.2 is broader (change composition vs specific reduction of oxygen).

      Further, they relate to different problems: II.1 to preserving specific ally wine, and II.2 to alcoholic beverages in general. So, they do not provide alternative solutions to the same problem, but only to related problems.

      Delete
    11. I believe that the fundamental idea is one of the claims cannot be a species of the second claim, hence they need to recite separate independent claims.

      I also believe that both claims relate to the same problem (preserving an alcoholic beverage, which includes wine), but provide alternative solutions to the problem.

      Thanks

      Delete
  15. I (respectfully) disagree with your answer 11.1.

    First, there is NO standardized size for a FOLDED yoga mat.

    The claim 11 is only “Textile bag for completely covering and for transporting yoga mats.”

    So, I’m OK that the textile bag must have dimensions greater than at least TWO yoga mats. (It’s indicated yoga matS in the claim)

    But the dimensions are not limited as long as the dimensions of the textile bag are greater than at least two yoga mats. For example, if we can a take a volume to simplify, the volume of the textile bag can be 0.2cm3, 1cm3, 2cm3, 10cm3, 50cm3… as long as they are greater than the volume of at least two folded yoga mats.

    In a general way, I think we can’t consider “not too small” as a limitation of the dimensions.

    (For example, with a claim : Bottle for water storage ; OK with this claim that the dimensions of the bottle have to be greater than a water molecule (and a water molecule has a “standardized” size) but for me it’s not a limitation of the dimensions of the bottle)

    So, the answer of 11.1 should be FALSE.

    ReplyDelete
  16. you already contradict yourseld when you said "as long as they are greater than the volume of at least two folded yoga mats". This already implies a limitation.

    Then, the answer is True

    ReplyDelete
    Replies
    1. With your reasoning, all product claims have a limitation.

      But I think we can't consider as a limitation if the dimensions can have all values.

      Moreover, the textile bag can have dimensions smaller than at least two yoga mats if the textile can be stretched.

      Delete
    2. "the textile bag can have dimensions smaller than at least two yoga mats if the textile can be stretched."

      There is no indication of stretchable textile/bags in the paper, so this argument does not seem valid - R.22(3) IPREE

      Further, your suggested stretchable bag still needs to satisfy a limitation as to is dimensions, i.e, the dimension that is can reach in stretched form.

      But I think we do agree that the wording of the statement and what it intended to test is not clear:
      i) If "a limitation of the dimensions of the textile bag" is taken as "any limitation of the dimensions" it is T (as, e.g., a A5-size, envelope-shaped, non-stretchable bag will clearly not be within the scope of the claim so that it does impose a limitation);
      ii) if "a limitation of the dimensions of the textile bag" is taken as "a well-defined limitation of the dimensions as the size of a rolled yoga mat is standardized" (whereas only its size if rolled-out is standardized, but not even its thickness), it is T;
      iii) whereas if "a limitation of the dimensions of the textile bag" is taken as "a ill-defined limitation of the dimensions as the size of a yoga mat ready for transport is not standardized and the range of dimensions is also quite undefined" it is F - it could be transported when rolled into a roll of undefined diameter, it could be folded into a rather flat object, ...
      iv) if "mats" is to be interpreted as "one mat at a time, but sequentially possible different mats", the interpretations of i-iii may also be different than if interpreted as "a plurality of mats at the same time"

      Delete
    3. Should this question be neutralized?

      Delete
    4. Examiner’s Report:

      “ 11.3 The features of claim 11 imply a limitation of the dimensions of the textile bag
      TRUE: A textile container for completely covering and for transporting a yoga mat re-
      quires certain minimum dimensions (Guidelines F-IV, 4.14.2). The dimensions of a yoga mat are mentioned in [001] of the description. For example, a textile case, which cannot be extended to one of the dimensions of a yoga mat mentioned in [001] of the description, would not be covered by claim11.”

      Delete
  17. Good morning,
    I have some doubts about question 17.4.

    My answer was T: D12 destroys the novelty of the subject-matter of claim II.4, according to the last sentence of paragraph 002.
    In particular, that last sentence, in my opinion, with the wording "is injected therein" discloses also what is stated in claim II.3. In fact, "therein" from my understanding is related to the headspace. Thus, to the injection of the gas in the headspace of the bottle.

    Here are the passages of D12 paragraph 002, for your convenience:
    We have additionally noted that a part of the injected carbon dioxide reaches
    the headspace of the bottle and pushes the air (and therefore also oxygen) contained therein out of the headspace of the bottle. Moreover, we found that wine can be better preserved when carbon dioxide is injected therein.

    Do you have any comments about it? I hope I explained my point clearly.
    Thank you

    ReplyDelete
    Replies
    1. Good morning,

      in my opinion, in D12 the gas (carbon dioxide) is directly injected into the wine, but not directly into the headspace of the bottle.

      Afterwards, the gas, which was already directly injected into the wine,reaches the headspace of the bottle by bubbling out of the wine and pushes the air (and therefore also the oxygen) contained therein out of the headspace of the bottle.

      Thus, in my opinion question 17.4 has to be answered with false (F).

      Delete
    2. Maria Caterina refers to the sentence: "we found that wine can be better preserved when carbon dioxide is injected therein". Therein being the head space.

      Her argument makes sense IMHO

      Delete
    3. I think it is clear that the carbon dioxied is injected directly into the wine and this is why paragraph [0002] states that "We have additionally noted that a part of the injected carbon dioxide reaches the headspace of the bottle and pushes the air". The whole document is about injecting co2 into the wine and I think by "therein", they meant the wine and not the headspace.

      Delete
    4. @Anonymous21 March 2022 at 15:30:
      Upon review of paragraph [002], I note that there are two statements:
      1) We have additionally noted that a part of the injected carbon dioxide (INJECTED INTO THE WINE) reaches the headspace of the bottle and pushes the air contained THEREIN out of the headspace".
      2) Moreover, we found that wine can be better preserved when carbon dioxide is injected THEREIN.

      Given that the previous sentence was discussing the headspace, it is confusing whether the last statement refers to the wine or to the headspace.


      Delete
    5. Does not "Moreover, we found that wine can be better preserved when carbon dioxide is injected THEREIN" mean "Moreover, we found that wine can be better preserved when carbon dioxide is injected INTO WINE"? That also matches the logic of the structure of [00] of D12: in the first sentence, they have "found that the same devices can be used to inject the carbon
      dioxide into wine", and they conclude that if they do so "wine can be better preserved when carbon dioxide is injected therein [into wine]" (next to the pleasant fizz).

      Without that context of the rest of the paragraph, the "wherein" could be considered ambiguous as it could then also refer to the end of the preceding sentence: "We have additionally noted that a part of the injected carbon dioxide reaches the headspace of the bottle and pushes the air (and therefore also oxygen) contained therein out of the HEADSPACE of the bottle. Moreover, we found that wine can be better preserved when carbon dioxide is injected THEREIN [thew headspace, rather than the wine]". In the context, I find it less/not ambiguous, and referring to the wine.

      Delete
  18. Does any one notes the translation error from the English version of the description of Part 3 into the German language, i.e. Poly-X, instead of Poly-Y, has been explicitly referred to the purpose of providing improved reduction of sweat generation on the body of the athlete while the athlete is in contact with the mat.

    Hence, in the German version of the description of Part 3, Poly-X has two purposes: not only (i) providing secure but reversible adhesion of the yoga mat onto the floor as a coating on the first face of the yoga mat, but additionally (ii) providing improved reduction of sweat generation on the body of the athlete while the athlete is in contact with the mat as being contained in the second face of the yoga mat.

    Consequently, is there a chance that question 15.1 will be neutralized due to unequal treatment?

    In fact, as a German native speaker, only due to said translation error, i answered question 15.1 with false (F).

    ReplyDelete
    Replies
    1. You mean [005]?

      [005] In an embodiment, the second face contains a material which reduces the
      generation of sweat on the athlete’s body. We have found that poly-Y is a highly suitable
      material, since it provides improved reduction of sweat generation on the body of the
      athlete while the athlete is in contact with the mat. Using our new special secret test
      protocol we established a sweat reduction value (SRV) of 100 for yoga mats made of
      poly-Y. Higher SRVs correspond to a higher level of sweat reduction. For yoga mats made
      of poly-X the SRV is 84, and for yoga mats made of poly-Z the SRV is 93. In an
      embodiment of the invention the yoga mat has an SRV of 90 or more.


      [005] In einer Ausführungsform enthält die zweite Seite ein Material, das die
      Schweißbildung im Körper des Sportlers reduziert. Wir haben festgestellt, dass Poly-X ein
      sehr geeignetes Material ist, denn es sorgt für eine verbesserte Reduzierung der
      Schweißbildung auf dem Körper des Sportlers, während der Sportler Kontakt zur Matte
      hat. Anhand unseres neuen speziellen geheimen Testprotokolls haben wir bei Yogamatten
      aus Poly-Y einen Schweißreduktionswert (SRW) von 100 ermittelt. Ein höherer SRW
      entspricht einem höheren Grad der Schweißreduktion. Bei Yogamatten aus Poly-X beträgt
      der SRW 84, und bei Yogamatten aus Poly-Z beträgt der SRW 93. In einer
      erfindungsgemäßen Ausführungsform hat die Yogamatte einen SRW von 90 oder mehr.

      Delete
    2. Do you think there is a chance of neutralizing question 15.1 due to unequal treatment? Besides, this might be an interesting point for an appeal, isn't it?

      Delete
    3. There would not be a need to appeal if they neutralize any statements that depend on it. Which seems reasonable to expect, as they did it with earlier differences between language versions. Just my five cents.

      Delete
  19. Q19.1:
    the question is a bit confusing. Deleting Argon from claim 7 would leave claim 9 (dependent on claim 7) to have unallowable intermediate generalization? claim 9 has to refer to Argon because [004] also said that Argon is preferred because it's heavier than air and can create the gas gap. Not all inert gas can be combined with the feature "opened bottle". The example of the inert gas Nitrogen which is lighter and will escape from an opened bottle can be found in D13.

    ReplyDelete
    Replies
    1. D13 states that it is possible to remove the oxygen from the headspace with nitrogen, the only difference is you need a larger amount of it compared to how much argon you would need, since it is not heavier than oxygen. It specifically says that this is done before the bottle is closed so this implies that the process occurs during an opened state.

      Looking at claim 8 and the document, it can be seen that argon is not essential to remove the oxygen from the headspace of the opened bottle, so I think the question is clear. If one of the claims containing argon also had a feature relating to the gas cap, I think you would be right because that feature would be considered to be unallowable intermediate generalisation as gas cap is inextricably linked to argon. I've marked false on that one aswell but it should not be confusing when you think about it.

      Delete
    2. Interesting point liz. Deleting argon from the claims would make claim 7 empty, so that claim II.9 would depend on II.2 and II.3 and thus read -when written out in full:

      II.2. Method for preserving an alcoholic beverage in a bottle, wherein the amount of oxygen in the headspace of the bottle is reduced,
      wherein an inert gas is injected directly into the headspace of the bottle,
      wherein the alcoholic beverage is wine and the wine is contained in an opened bottle.

      Of the inert gasses, "Argon is the most preferred inert gas, because it is heavier than oxygen and heavier than air, so that a layer of argon creates a “gas cap” on top of the wine level." and indeed D13 [002] says "It is known by the skilled person that nitrogen is lighter than oxygen", so that it cannot create a cap.
      I agree that the feature of original claim II.9 could be read to refer to the "gas cap".

      However, [003] also mentions inert gasses and seems to give proper basis in view of the "A preferred other gas is an inert gas. This is achieved by injecting the other gas (e.g. the inert gas) into the headspace. The injected gas pushes the air out of the headspace".

      What are your thoughts?

      Delete
    3. It doesn't have to be argon in the case of opened bottle, this is disclosed in the specification and there is basis for other gases. Moreover, claim 8 would not make any sense if it did. Argon is preferred because of the gas cap created thereby, however none of the claims directly refer to said gas cap and the gas cap isn't required to push the air out of the opened bottle, so it seems fine to me. As mentioned above, this argument would only be valid if the gas cap was also mentioned in the claims including argon.

      Delete
    4. Thank you @Roel. I understand your reasoning. It seems to me a confusion created from linguistic uncertainty (at least in the English version):
      The feature "opened bottle" in claim 9 can read with different meaning, either bottle once opened or bottle remain opened. Interpretation of bottle once opened can be derived from [003]. There is no clear support from the description for the interpretation of "bottle remain opened" but since the claims are filed with the application, the claims also forms as part of the application as filed.
      When analyzing if the intermediate generalization is allowable, one of the criteria is to judge if the changed feature is related or inextricably linked to other features in the claims incl. dependent claims. For the interpretation of "bottle once opened", the intermediate generalization is allowable, as you explained. For the interpretation of "bottle remain opened", it can be read that there is a link between the opened bottle and Argon where "gas cap" is only possible with Argon for an opened bottle. Especially with regard to the statement about Nitrogen by the skilled person in D13, the intermediate generalization seems not proper and would not be allowable.
      I've checked a dictionary where it says that "opened" can be: not sealed or having been unsealed. Since there is a plausible scope of the claim and the combination of the features is only in claim 9 as a specific embodiment I would choose F even regardless of D13. As it is stated in the CLBoA II-E, 1.9: The board in T 962/98 held that an intermediate generalisation was admissible only if the skilled person could recognise without any doubt from the application as filed that those characteristics were not closely related to the other characteristics of the working example and applied directly and unambiguously to the more general context (often cited, see e.g. T 144/08, T 313/09, T 879/09, T 2185/10, T 2489/13).
      Looking forward to your opinion and thanks for the interesting discussion!

      Delete
  20. I think 20.2 can also be TRUE if we consider that D13 is suitable for preserving wine in a bottle (see the previous discussions about limitation or not about "for preserving wine in a bottle").

    ReplyDelete
  21. I don't understand your answer about 16.4.
    In the description, it's clearly indicated that the invention is the same for wine or beer (about preservation).
    In D13, it's clearly indicated “Clearly, this method can be applied to many types of liquid, such as perfumes and beer”.
    So D13 is suitable for preserving wine.

    Claim II.1. can be read like this: Method wherein the composition of a gas in the headspace of the bottle is modified.

    So, D13 destroys the novelty of the subject-matter of claim II.1.
    The answer should be True.

    ReplyDelete
    Replies
    1. The prior art needs to be taken as it is for novelty - generalizations, equivalents, similarities are not disclosed, not explicitly, not implicitly.

      Also, the claim must be taken is at is. You rephrased the claim into something different.

      So, for lack of novelty, comparable (beer vs wine) is not good enough - that would be a matter of inventive step.

      Delete
    2. For example in the exam 2014 (even if I know Exams are not legal texts), a claim was "Bottle for a liquid soap dispenser, the bottle having a base for touching a tube of a liquid soap dispenser".

      The documents from the prior art were with no tube or with a tube very short (a tube that didn't touch the base). And all the documents from the prior art destroyed the novelty.

      Because the claim could be read like this: “Bottle having a base”.
      (Any base of a bottle is suitable for touching a tube and the claim is not limited to comprise a tube)

      The same reasoning can be applied for the claim "Method for preserving wine in a bottle". This method is not limited to wine. And it's clearly indicated that D13 is suitable for preserving wine.

      So, D13 destroys the novelty of the subject-matter of claim II.1.
      The answer should be True.

      Delete
  22. Dario, the 2014 exam paper that you are talking about is a product claim. This is a method claim.

    You cannot use the same guidelines to determine the limiting feature of the method claim that you use for a product claim.

    For Guidelines on what is a limiting feature of a method claim, please see F-IV, 4.13.3.

    ReplyDelete
    Replies
    1. And in the guidelines, it's clearly indicated:

      "On the other hand, where the purpose merely states a technical effect which inevitably arises when carrying out the other remaining steps of the claimed method and is thus inherent in those steps, this technical effect has no limiting effect on the subject-matter of the claim.

      For example, a method claim concerning the application of a particular surface active agent to a specified absorbent product and defining its purpose as "for reducing malodor" in terms of an intended technical effect is anticipated by a prior-art document describing a method having such suitability "for reducing malodor" although not mentioning the specific use (see T 1931/14 and T 304/08)."

      Delete
    2. @Asim, in your opinion, in which of the three categories reported in F-IV, 4.13.3., the present claim falls into:
      1) a method for remelting galvanic layers,
      2) a method for manufacturing a product, or
      3) a method for reducing malodor

      I look forward to hearing your views on this.

      Thanks

      Delete
    3. great!!!!!! another exam

      Delete
  23. @Dario, yes I know. I was just making sure that you get your legal basis correct. Claims pertaining to “Method for” and “Product for” are treated differently. Now you know that the claims pertaining to “method for” can be placed under two categories, one where the purpose is a limiting feature and another where the purpose is not a limiting feature.

    Regarding your argument that the wine claim belongs to the category in which the purpose is not a limiting feature, I somewhat agree, and I already raised this point on Friday (see discussion, above) but I am not “absolutely” sure if I am correct. Roel seems to also disagree with me.

    It’s not a well-established principle in real life supported by, for example, a G decision.

    ReplyDelete
  24. To Question 20.2:

    Question 20.2 is an ill-posed question.

    The given set of claims includes two independent claims, namely claim II.1 and claim II.2.

    There are two inventions defined by the two independent claims II.1 and II.2.

    However, according to Question 20.2, it is not directly and unambiguously stated, whether D13 is to be regarded as the closest prior art in view of claim II.1 or claim II.2.

    Therefore, Question 20.2 can be so interpreted, that D13 can be considered to be the closest prior art in view of independent claim II.2, together with the corresponding dependent claims II.3 to II.9.

    According to Guideline for the Examination in the EPO (GL – G–VII, 5.2), one should study "the distinguishing feature(s)" between the claimed invention and the closest prior art in order to establish and formulate the objective technical problem.

    In this case, D13 is considered to be the closest prior art in view of claim II.9. Due to its dependency, claim II.9 includes all the features of claim II.2, claim II.3, claim II.7 and claim II.9.

    More specifically, claim II.9 defines a method for preserving an alcoholic beverage in a bottle, wherein the amount of oxygen in the headspace of the bottle is reduced, wherein an inert gas is injected directly into the headspace of the bottle, wherein the inert gas is argon, and wherein the alcoholic beverage is wine and the wine is contained in an opened bottle.

    D13 is directed to a method for preserving a liquid from oxidation.

    D13 discloses all the features of claim II.9 except “wine” and “argon”.

    The distinguishing features “wine” and “argon” can be considered in two separate partial problems to evaluate an inventive step of claim II.9 in view of D13.

    In respect of the distinguishing feature “wine”, according to paragraph [003] of D13, the method for preserving liquids can be applied to many types of liquid, such as perfumes and beer. Therefore, the skilled person would obviously apply the method to preserve wine from oxidation.

    In respect of the distinguishing feature “argon”, D13 discloses a method of injecting nitrogen into the headspace of the wine. Since nitrogen is lighter than oxygen, sufficient nitrogen, for instance 20 times of the volume of the headspace, has to be injected so as to extract oxygen, thereby preventing the wine from being oxidised by the contact with oxygen.

    In contrast to D13, claim II. 9 defines a method for preserving the wine contained in an opened bottle by injecting argon directly into a headspace of the wine.

    Since argon is heavier than oxygen as well as air, a layer of argon can create a “gas cap” on top of the wine level. Such “gas cap” isolates the wine level from oxygen and can therefore prevent the oxidation which occurs, if the wine is in contact with oxygen.

    Therefore, the objective technical problem to be solved by the invention defined by claim II.9 in view of D13 can be seen as how to reduce the amount of gas required to remove air from above the wine level.

    Hence, it is also correct to answer Question 20.2 with a “True”.

    ReplyDelete
    Replies
    1. I agree that the CPA can only be defined relative to a specific claim - which specification was missing in all CPA statements. And that it can be different for different claims.

      Be careful though by just comparing features. CPA is not determined by correspondence of one feature that seems to be the nature of the invention, but by the closest "combination of features" = embodiment. Some "features" are therein more important than others: the features that determine the field of the claim as a whole, and the purpose of the claim as a whole (i.e., not the purpose of the distinguishing feature - the latter can only be derived AFTER we have thew CPA, and the latter will set the objective technical problem).

      Delete
    2. In my view, "wine" in II.9 is a major feature that sets the scope of application of the method, and thereby requires the CPA to relate to preserve wine, and not just any other alcoholic beverage (unless there is no prior art with wine, then the field/purpose can be broader or neighboring, such as beer). That is one of the reasons why the CPA varies per claim.

      Delete
    3. You seem to consider "wine" to be more important than the other distinguishing feature "argon". I do not agree with this. Given the invention as a whole, the core inventive idea lies in "injection an inert gas which is heavier than oxygen". Based on your opinion, I am curious about what is the technical effect of "wine". Oxidation confronts also other alcoholic beverage in an opened bottle, as is the case of wine.

      Delete
    4. Roel, I can see why you think that wine should be disclosed in the closest prior art (in order for a document to be the CPA). However, question 20.2 says to *assume* that D13 is the CPA, so whether that is a correct choice is immaterial. So, assuming that D13 is the CPA, I must say I find RL's reasoning regarding a partial problem approach convincing, and that the given OTP appears to be a valid OTP for the second partial problem.

      Personally, I simply overlooked that the OTP in the question explicitly referred to wine, while D13 does not explicitly disclose wine. But I do think that is at least in part because we had to construe the claim for which D13 could reasonably be assumed the CPA ourselves. For example, claim II.2 is not even new over D13, so assuming D13 is the CPA does not make sense in that case.

      Delete
    5. @Gerben: claims 1, 8 and 9 are about preserving wine; claims 2-7 are about preserving an alcoholic beverage - for claims 2-7 the CPA does not need to relate to wine.

      Delete
    6. @RL: Indeed, as the claim is all about preserving wine. That is also what I would choose to drink. Argon is just a way to achieve it, but I as consumer do not care about the way how - as long as it is preserved and remains wine.
      How would you react in a restaurant when you get beer while you ordered a nice bottle of red wine?

      Delete
    7. @Roel, Yes, my first statement should have included "when considering II.9 (or a similar claim related to preserving wine using argon)"

      However, I am not sure I understand what point you are trying to make? If the claim (e.g., claim II.9) is about preserving wine, the CPA should typically be about preserving wine, sure - but given that the statement assumes D13 is the CPA, the question of whether or not D13 is a correct choice is moot. Or are you arguing that the question cannot relate to claim II.9, as in that case, the selection of CPA would be wrong?

      Assuming that statement 20.2 relates to claim II.9, and given that D13 is assumed to be the CPA, how would you phrase the OTP?

      Delete
    8. Gerben, spot on. I totally agree with you. The question explicitly states that it is assumed that D13 is the closet prior art and so there is no argument that D13 has no wine and thus, is not a suitable CPA. It’s simply moot.

      Delete
  25. To Question 20.3:

    According to Guideline for the Examination in the EPO (GL – G–VII, 5.1), the closest prior art is that which in one single reference discloses the combination of features which constitutes the most promising starting point for a development leading to the invention.

    The invention lies in injection of an inert air into a headspace of an alcoholic beverage (e. g. wine) contained in an opened bottle to remove air and also oxygen from the bottle. Thereby, the oxidation of the wine can be prevented.

    D11 discloses a vacuum pump that is used to extract air or oxygen from an opened bottle in order to preserve the wine contained therein.

    The vacuum pump is a one-way valve, through which air or oxygen can be extracted. In other words, no gas can be injected into the headspace through the vacuum pump.

    In contrast to D11, both D12 and D13 disclose injecting gas into the opened bottle.

    Since the injection of an inert air into the headspace of the wine is an essential feature of the invention, the one-way valve disclosed in D11 would lead the skilled person away from the invention.

    Thus, D11 cannot be considered to be the closest prior art and Question 20.3 can be answered with a “True”.

    ReplyDelete
    Replies
    1. Your argument relates to claim II.3.
      But claims II.1 and II.2 do not require injection of an inert gas, so your argument does not hold for those two claims.

      Your argumentation shows that, without a reference to specific claim in the question/statement, the statement is ambiguous and unclear, and cannot be unambiguously answered.

      Other opinions?

      Delete
    2. I also think that questions 20.2 and 20.3 are unclear.

      In this case, each claim should be examined for its inventive step. In practice, do the examiner also so.


      II. 1 and II.2 were seemed to be defined by the result to be achieved. Such definition is not allowed (F-IV, 4.10), so features as such should not be considered for patentability evaluation.

      Therefore I agree with the argumentation of RL.
      II. 3 and II. 9 can also be drawn into Betratung, so that D11 cannot be considered the closest SdT to II. 3, and D13 can be considered the closest Sdt to II.9. The statement 20. 2 and 20.3 can also be regarded as true.

      Delete
    3. I agree that it was very confusing that it was not specified which claim(s) the statements in question 20 referred to. I have a feeling that it was the intention to provide a single new claim that is at least somewhat reasonable, but that somehow this amended claim was not included in the question. I also agree that whether statement 20.3 is True or False depends on the claim which is considered, and that hence the question is not unambiguous, contrary to what may be expected in a multiple-choice exam.

      Delete
  26. Could you please explain a little more why question 13 - "The subject-matter of claim 10 is novel over D1" is false? In my view, D1 refers to mats made completly out of Poly-Y, while claim 10 actually refers to the second side. D1 is also silent about sweat reduction. Thank you!

    ReplyDelete
    Replies
    1. Claim 10 defines that the second face contains a material, wherein the material is such that upon contact with the athlete’s body it reduces the generation of sweat on the athlete’s body. The fact that the material is Poly-Y does not represent a limiting feature for claim 10 but only an optional one. Moreover, document D1 describes that the yoga mat is completely made of Poly-Y. Poly-Y is known, for instance from paragraph [005] of the European patent application, as a material that provides improved reduction of sweat generation on the body of the athlete while the athlete is in contact with the yoga mat. Accordingly, the yoga mat completely made of Poly-Y as characterized in document D1 implicitly discloses that the material is such that upon contact with the athlete’s body it reduces the generation of sweat on the athlete’s body.

      Delete
  27. 20.1 and 20.2 can also be True (and should be True...).
    Question 20 is very unclear because we don't know if we have to choose ourselves the claims, if we can combine ourselves some claims, if we can amend ourselves the claims (of course supported in the description) or if we can take the common meaning of the description... Because in the questions, it’s only indicated "If D12/D13 is selected as the closest prior art".

    Also if we consider that the wine is suitable for D13 (see the discussions about that in this blog), 20.2 can also be True for this reason.

    ReplyDelete
  28. I can now enroll to the main examination and the option to enroll to the pre-exam is grayed out for me, which was working a few days ago. Not sure if this means I've passed, does anyone else's my-eqe look like this?

    ReplyDelete
    Replies
    1. Same for me. I guess we have passed it

      Delete
    2. ditto. Here's hoping! glad to see I'm not the only one looking up myEQE! Hopefully not so long to go till we have official notice. Given the timing of Easter and in light of the 2021 procedure it would be nice to have the letter by next Thursday.

      Delete
    3. Same for me I hope this means good news !

      Delete
  29. Art. 123(3) EPC6 April 2022 at 09:35

    Have I passed? I can now enrol to the main examination too....What gives?

    ReplyDelete
  30. They answered to me results for PreExam will be ready mid-April.

    ReplyDelete
    Replies
    1. Rules 70(2) and 70a(2) EPC7 April 2022 at 11:41

      Vague! Nevertheless...some time...next week.....?!

      Delete
    2. Is there anyone who doesn't have theirs greyed out? Or are they assuming that everybody has passed until proven wrong? The Main Exam opened on the 4 April.

      I so wish this means a pass. I hope they will not un-grey the pre-EQE once the results come out.

      Any insider's view on this?

      Delete
    3. The Cool Attorney7 April 2022 at 15:04

      Anyone? Anyone?

      Delete
    4. I vaguely remember both being open to me last year prior to having registered for the pre-exam so while I am hopeful that not being eligible to re-enrol for the pre-exam means a pass, I don't think it's something that can be relied on for making any decisions.

      It is interesting how they opened the main exam registration on April 4th. Only a very, very small number of candidates will or can use this facility at the moment. The pre-exam results aren't out, nor do the main exam candidates who will resit know what they're doing next year. That leaves only those who postponed the main exam or a resit of part thereof.

      Assuming I have passed, I am dreading the wait next year as the pre-wait has been horrible so far!

      Delete
    5. Refreshing MyEQE... no news yet...

      Delete
    6. I've just received a mail from the Secretariat that a new document is available on myEQE, but disappointment, it is just a confirmation of my participation at the exam...

      Delete
    7. Article 123(2)7 April 2022 at 18:42

      Me too! I wonder how long 'shortly'is....

      Delete
    8. My heart sank once I realised. But I guess this means the wiseflow data is all transferred and they are just dotting is and crossing ts!

      Delete
    9. the wealthy patent licensor8 April 2022 at 09:07

      My fellow humanoids..what does "shortly" mean...?

      Delete
    10. Not even aliens know...

      Delete
    11. the wealthy patent licensor8 April 2022 at 09:14

      What are aliens to you?

      Delete
    12. çilekeş tosbağa8 April 2022 at 09:28

      we'll probably get the results next wednesday or so

      Delete
    13. the skeptical patent agent8 April 2022 at 09:48

      How likely is it that we'll get the results today..dear friendos?

      Delete
    14. Please check your crystal ball, and then tell us ;)

      Delete
    15. Frantic myEQE refresher8 April 2022 at 10:16

      I am really hoping it is today!!

      Delete
    16. they've said mid april, so...

      Delete
    17. Selfish patent-work loving humanoid8 April 2022 at 10:32

      what's the definition of..."mid april"...?

      Delete
    18. Last year, 'shortly' was the same or the following day I think. So Fingers crossed for today?

      Delete
    19. Bunny side down8 April 2022 at 11:55

      Is exactitude in terms of when the results would be released too much to ask?

      Delete
    20. @the wealthy patent licensor and all others:

      We now know: "shorty" meant today!

      Hope you are all satisfied with your results, and wish that I can congratulate you!

      Delete
  31. Solitary Zen practitioner7 April 2022 at 16:00

    Is that so?

    ReplyDelete
  32. the restuls are out on my eqe

    ReplyDelete
  33. Results online! I have 8 more points than I've calculated in view of the deltapatents answers. So I guess they cancelled some questions.

    ReplyDelete
    Replies
    1. I hate question 208 April 2022 at 14:27

      Same here, I earned about 7 points higher than I'd calculated. I'd guess that the question regarding CPA assessment is neutralized

      Delete
    2. But not only Q20 imo.

      Delete
    3. yeah, maybe the wine thing aswell?

      Delete
  34. Congratulations for all that passed!!!

    The Examiner's Report is not yet available, so if you did not score 100 marks, you still need a bit of patience to know where you lost some - and where we may have given a different answer...

    ReplyDelete
    Replies
    1. ..and to know what has been neutralized and for what reasons.

      Delete
    2. Please feel invited to post any comments in your results (was it more or less as expected, what do you conclude from it, ...) to our new blog post:

      http://pre-exam.blogspot.com/2022/04/pre-exam-2022-congratulations-to-all.html

      Delete
  35. Had a lovely time interacting with you all! A special shout-out to you, Dr. Woudenberg! See you all next year in the aftermath of the 2023 main exam! Svaha...

    ReplyDelete
    Replies
    1. Also hope to see some of you already during your preparation for the 2023 exam! ;)

      Delete
  36. The Examiner’s Report is now available on the EQE Compendium pages https://www.epo.org/learning/eqe/compendium/preexamination.html

    It says to part 3:

    “Remark: There is a translation error in the German version [005] of the description, second line. It should read Poly-Y and not Poly-X.
    Due to this translation error the Examination Board decided to award full marks to all candidates for questions 11 to 15.”

    Also, w.r.t. Question 20, it says:

    “Since the following sentence was missing on WiseFlow:

    “Assume in the following that inventive step has to be assessed in respect of claim II.9.”

    the Examination Board decided to award full marks to all candidates for question 20.”

    ReplyDelete

Post a Comment